LSAT and Law School Admissions Forum

Get expert LSAT preparation and law school admissions advice from PowerScore Test Preparation.

 Administrator
PowerScore Staff
  • PowerScore Staff
  • Posts: 8926
  • Joined: Feb 02, 2011
|
#63990
Complete Question Explanation

Assumption. The correct answer choice is (D)

Here the economist presents a causal argument, which we might recognize based on the use of
words such as “result” and “cause.” Basically, the economist says that factors such as high-priced
goods and services cause consumers to be reluctant to spend, and this in turn causes our economy’s
weakness. The situation is made worse by a lower average income, so the author concludes that
lower income taxes, though not a perfect solution, would cause the economy to improve.

How would lower income taxes cure the reluctance to spend? The author must assume that lower
income taxes, meaning more money going back to consumers, would cause consumers to be less
reluctant to spend.

The stimulus is followed by an assumption question; if lower taxes will reduce reluctance to spend,
the presumption must be that the money consumers don’t spend on taxes will be spent consuming
(rather than saving or investing, for example).
Answer choice (A): This assumption is not required by the argument in the stimulus, because the
conclusion is that the economy would improve, and this does not mean that prices would necessarily
need to decrease.

Answer choice (B): This answer choice is incorrect for the same basic reason that answer choice
(A) above is incorrect: Since the economist makes it clear that less spending reluctance will help
the economy, the conclusion (that the economy will improve) does not require that average incomes
increase when consumer spending does so.

Answer choice (C): The author’s argument only deals with what will happen if income taxes are
lowered; there is no reference to, or assumption about, what will happen if income taxes are not
lowered, so this answer choice cannot be correct.

Answer choice (D): This is the correct answer choice. This choice presents exactly the Supporter
Assumption that we seek, and the one which allows for the economist’s conclusion to be properly
drawn. If lower income taxes will cause consumers to be less reluctant to spend, then we can add this
supporter assumption to the premises from the stimulus, and link the rogue elements to arrive at the
economist’s conclusion.

..... ..... ..... ..... ..... ..... C ..... ..... ..... E

..... Premise: ..... less reluctance to spend :arrow: help economy
..... ..... +
Supporter Assumption: lower Income taxes :arrow: less reluctance to spend
Conclusion: lower income taxes :arrow: less reluctance to spend :arrow: help economy

Answer choice (E): Like the other incorrect answer choices above, this choice is not required of the
author’s argument: regardless of any effects on government spending, the author only concludes that
lowering the income tax will improve the economy.
 Bahar
  • Posts: 8
  • Joined: Jun 27, 2018
|
#49316
How I diagram this question is :
pre: reluctant to spend------> Weaken economy
Contro positive : improve economy ------->less reluctant


conclusion: lower tax ------>economy improve



the gap is between less reluctant and lower tax: if we have: lower tax------>less reluctant, then still we are mistaken the reverse from less reluctant we cant conclude improve economy ... what we need is: less reluctant----->lower tax.

Can somebody tell me where I am wrong?
User avatar
 Jonathan Evans
PowerScore Staff
  • PowerScore Staff
  • Posts: 726
  • Joined: Jun 09, 2016
|
#49545
Hi, Bahar!

Good analysis! This argument uses a great deal of causal reasoning. Since it is so heavy on cause/effect, it does not lend itself as well to conditional diagramming, with the exception of the conclusion, which is conditional. We could break it down as follows:
  1. Reluctance to spend causes economic weakness.
  2. High priced goods/services causes reluctance to spend.
  3. Average income is low causes reluctance to spend.
  4. Conclusion: Lower income taxes :arrow: improved economy
How do we get to an improved economy? An improved economy is the negation of economic weakness. If we wish to eliminate economic weakness, what would we need to do? We know that reluctance to spend causes economic weakness. Therefore, if we wish to eliminate economic weakness, one way would be to decrease reluctance to spend. With less reluctance to spend (more willingness to spend) we could expect less economic weakness (an improved economy). We know this from premise (1) above. What does that author seem to think is connected to getting rid of reluctance to spend? The author attempts to make a connection between income taxes and how willing people are to spend money.

The author's assumption is that if you lower income taxes, people will be less reluctant to spend money. This is the gap that is identified in answer choice (D). Do the Assumption Negation test. What would happen if people were not less reluctant to spend money, even if income taxes were lowered? Does the author's argument make sense anymore? No, why would the author want to lower income taxes if there were no connection with willingness to spend money. The argument would be nonsensical. Therefore this is the assumption.

I hope this helps!
 Bahar
  • Posts: 8
  • Joined: Jun 27, 2018
|
#49747
Thanks for explanations.
So, How we know that we shouldn't use cause/effect for this kind of questions?
 Adam Tyson
PowerScore Staff
  • PowerScore Staff
  • Posts: 5162
  • Joined: Apr 14, 2011
|
#49993
That's a tough question, Bahar, because it really depends on a few factors. When arguments combine causal and conditional reasoning, we may get confused about which to apply, and that's part of why they do that.

In general, focus your efforts on the conclusion. A series of conditional premises might be followed by a causal conclusion, and a causal attack might be the right way to deal with that. Here we have causal premises and a conditional conclusion, so conditional tools might be the best choice for dealing with that conclusion.

As Jonathan says, we can always use a conditional approach to deal with causal arguments, and this is because causal arguments always have an underlying conditional relationship, which is "if the cause occurs, then the effect must occur."

Ultimately, it's about what you are trying to accomplish based on the question stem that will dictate the best approach. Since this is an Assumption question, we are likely looking at a gap between the premises and the conclusion, and with the conclusion being conditional, it's likely that a conditional assumption will best fill that gap. That, and the fact that in the premises the author gave us causal claims as premises, which we are typically to accept as true, rather than giving us correlations and then making subordinate conclusions about causality.

Great question, and fertile ground for a lot of trouble for LSAT takers! When faced with a mix of both, I would suggest you start by analyzing the question stem and the conclusion, and see where that takes you.

Good luck!
User avatar
 JNSIWL24
  • Posts: 14
  • Joined: May 10, 2023
|
#106271
Hi
I want to be crystal clear. The explanation indicates:

Supporter assumption (d): A—> B Lower income taxes —->less reluctance to spend

Premise 1 of stimulus: B—>C Less reluctance to spend —> improved economy
________________________________________________________________________
Conclusion: A—> C Lower income taxes —-> improved economy

In order to diagram premise 1, we diagrammed “reluctance to spend” as “less reluctance to spend.” When “reluctance—>economy’s weakness” is negated, mistaken errors occur. But, when “economy weakness” is negated we arrive at “improved economy,” which seems ok as a logical equivalent. I feel a slight uneasiness with “less reluctance” because I see “reluctance to spend” in the stimulus.

Please advise.
User avatar
 Jeff Wren
PowerScore Staff
  • PowerScore Staff
  • Posts: 413
  • Joined: Oct 19, 2022
|
#106303
Hi JNSIWL,

You're correct that the argument doesn't directly state that "less reluctance to spend" causes "improved economy."

The actual premise as worded in the argument is that "consumers' continued reluctance to spend" directly caused "our economy's weakness."

However, because this is a causal statement rather than a conditional statement, we can infer (and the argument assumes) that (all other things being equal) decreasing the cause would decrease the effect. (Notice that this causal statement appears in the premise rather than the conclusion, so we accept it as true for the purposes of analyzing the logic of the argument.)

This is not an error in conditional reasoning that we refer to as a Mistaken Negation. For conditional statements, you would NOT want to assume that the sufficient condition not occurring means that the necessary condition does not occur (i.e. a Mistaken Negation) because the necessary can occur without the sufficient condition.
User avatar
 JNSIWL24
  • Posts: 14
  • Joined: May 10, 2023
|
#106309
Thanks so much Jeff!! It helps a lot and I see your point!!

Get the most out of your LSAT Prep Plus subscription.

Analyze and track your performance with our Testing and Analytics Package.